Difference between revisions of "1985 IMO Problems"

(Day I)
m (= Problem 2)
 
(One intermediate revision by one other user not shown)
Line 9: Line 9:
 
[[1985 IMO Problems/Problem 1 | Solution]]
 
[[1985 IMO Problems/Problem 1 | Solution]]
  
=== Problem 2 ==
+
=== Problem 2 ===
  
 
Let <math>n</math> and <math>k</math> be given relatively prime natural numbers, <math>k < n</math>.  Each number in the set <math>M = \{ 1,2, \ldots , n-1 \} </math> is colored either blue or white.  It is given that
 
Let <math>n</math> and <math>k</math> be given relatively prime natural numbers, <math>k < n</math>.  Each number in the set <math>M = \{ 1,2, \ldots , n-1 \} </math> is colored either blue or white.  It is given that
Line 65: Line 65:
 
* [[1985 IMO]]
 
* [[1985 IMO]]
 
* [http://www.artofproblemsolving.com/Forum/resources.php?c=1&cid=16&year=1985 IMO 1985 problems on the Resources page]
 
* [http://www.artofproblemsolving.com/Forum/resources.php?c=1&cid=16&year=1985 IMO 1985 problems on the Resources page]
 +
* [[IMO Problems and Solutions, with authors]]
 +
* [[Mathematics competition resources]] {{IMO box|year=1985|before=[[1984 IMO]]|after=[[1986 IMO]]}}

Latest revision as of 03:00, 29 March 2021

Problems of the 26th IMO Finland.

Day I

Problem 1

A circle has center on the side $AB$ of the cyclic quadrilateral $ABCD$. The other three sides are tangent to the circle. Prove that $AD + BC = AB$.

Solution

Problem 2

Let $n$ and $k$ be given relatively prime natural numbers, $k < n$. Each number in the set $M = \{ 1,2, \ldots , n-1 \}$ is colored either blue or white. It is given that

(i) for each $i \in M$, both $i$ and $n-i$ have the same color;

(ii) for each $i \in M, i \neq k$, both $i$ and $|i-k|$ have the same color.

Prove that all the numbers in $M$ have the same color.

Solution

Problem 3

For any polynomial $P(x) = a_0 + a_1 x + \cdots + a_k x^k$ with integer coefficients, the number of coefficients which are odd is denoted by $w(P)$. For $i = 0, 1, \ldots$, let $Q_i (x) = (1+x)^i$. Prove that if $i_1, i_2, \ldots , i_n$ are integers such that $0 \leq i_1 < i_2 < \cdots < i_n$, then

$w(Q_{i_1} + Q_{i_2} + \cdots + Q_{i_n}) \ge w(Q_{i_1})$.

Solution

Day II

Problem 4

Given a set $M$ of $1985$ distinct positive integers, none of which has a prime divisor greater than $23$, prove that $M$ contains a subset of $4$ elements whose product is the $4$th power of an integer.

Solution

Problem 5

A circle with center $O$ passes through the vertices $A$ and $C$ of the triangle $ABC$ and intersects the segments $AB$ and $BC$ again at distinct points $K$ and $N$ respectively. Let $M$ be the point of intersection of the circumcircles of triangles $ABC$ and $KBN$ (apart from $B$). Prove that $\angle OMB = 90^{\circ}$.

Solution

Problem 6

For every real number $x_1$, construct the sequence $x_1,x_2,\ldots$ by setting:

$x_{n + 1} = x_n(x_n + {1\over n}).$

Prove that there exists exactly one value of $x_1$ which gives $0 < x_n < x_{n + 1} < 1$ for all $n$.

Solution

Resources

1985 IMO (Problems) • Resources
Preceded by
1984 IMO
1 2 3 4 5 6 Followed by
1986 IMO
All IMO Problems and Solutions